LSAT--Columnist

This topic has expert replies
Legendary Member
Posts: 549
Joined: Wed Jan 06, 2010 7:00 am
Thanked: 16 times
Followed by:3 members

LSAT--Columnist

by ssgmatter » Sat Apr 03, 2010 1:36 am
Columnist: Much of North America and western
Europe is more heavily forested and has less acid
rain and better air quality now than five decades
ago. Though this may be due largely to policies
advocated by environmentalists, it nonetheless
lends credibility to the claims of people who
reject predictions of imminent ecological doom
and argue that environmental policies that
excessively restrict the use of natural resources
may diminish the wealth necessary to adopt and
sustain the policies that brought about these
improvements.
Which one of the following, if true, most strengthens
the columnist's reasoning?
(A) Nations sustain their wealth largely through
industrial use of the natural resources found
within their boundaries.
(B) The more advanced the technology used in a
nation's industries, the greater is that nation's
ability to devote a portion of its resources to
social programs.
(C) A majority of ecological disasters arise from
causes that are beyond human control.
(D) If a compromise between the proponents of
economic growth and the environmentalists had
been enacted rather than the current policies,
the environment would have seen significantly
less improvement.
(E) The concern demonstrated by a nation for the
health and integrity of its natural ecosystems
leads to an increase in that nation's wealth.

User avatar
Legendary Member
Posts: 748
Joined: Sun Jan 31, 2010 7:54 am
Thanked: 46 times
Followed by:3 members

by outreach » Sat Apr 03, 2010 4:59 am
A. Correct. Last line states "restrict the use of natural resources may diminish the wealth necessary...." and A stregthens this.
B. arg does not focus on advanced tech and social programs. so resources devoted to social prog is not releavnt
C.Out of scope
D. this weakens the arg
E. this weakens the arg
-------------------------------------
--------------------------------------
General blog
https://amarnaik.wordpress.com
MBA blog
https://amarrnaik.blocked/

Legendary Member
Posts: 549
Joined: Wed Jan 06, 2010 7:00 am
Thanked: 16 times
Followed by:3 members

by ssgmatter » Sat Apr 03, 2010 5:20 am
not clear can you explain it in more details....by stating the conclusion

Regards
PHil

User avatar
Site Admin
Posts: 2567
Joined: Thu Jan 01, 2009 10:05 am
Thanked: 712 times
Followed by:550 members
GMAT Score:770

by DanaJ » Sat Apr 03, 2010 5:58 am
Received a PM.

The reasoning goes as follows:
- there have been notable improvements in the quality of the environment
- this supports the idea that you still need to have some economical activity to support the costs associated with environmental improvements

The conclusion here is that wealth generated by the use of resources can be used for improving the environment.

A is spot on. If the wealth of a nation is generated by the use of its resources, then we actually do need to use these resources or else we can't pay for the environmental improvements.

B may be true, but there's no mentioning of the costs. What if technology is not that expensive (I know it sounds silly, but you need to take into account only what's given and avoid making assumptions)? Besides, social programs do not necessarily mean environmental programs.

C - could be true, but does not affect the argument of wealth and environmental improvements.

D - could be true, but does not reference the conclusion, i.e. no mentioning of wealth.

E - this is actually the "reverse argument" option. This option states that concerns for the environment lead to greater wealth. The conclusion, however, has it the other way around. So this option would weaken the argument because it says that the reasoning is flawed (i.e. backwards).

Legendary Member
Posts: 549
Joined: Wed Jan 06, 2010 7:00 am
Thanked: 16 times
Followed by:3 members

by ssgmatter » Sat Apr 03, 2010 8:52 pm
Thankyou Dana....It makes sense now!

Warm Regards,
Phil

Senior | Next Rank: 100 Posts
Posts: 37
Joined: Tue Feb 02, 2010 12:24 pm

by manjus_mailme » Mon Nov 15, 2010 4:10 pm
DanaJ wrote:Received a PM.

The reasoning goes as follows:
- there have been notable improvements in the quality of the environment
- this supports the idea that you still need to have some economical activity to support the costs associated with environmental improvements

The conclusion here is that wealth generated by the use of resources can be used for improving the environment.

A is spot on. If the wealth of a nation is generated by the use of its resources, then we actually do need to use these resources or else we can't pay for the environmental improvements.

B may be true, but there's no mentioning of the costs. What if technology is not that expensive (I know it sounds silly, but you need to take into account only what's given and avoid making assumptions)? Besides, social programs do not necessarily mean environmental programs.

C - could be true, but does not affect the argument of wealth and environmental improvements.

D - could be true, but does not reference the conclusion, i.e. no mentioning of wealth.

E - this is actually the "reverse argument" option. This option states that concerns for the environment lead to greater wealth. The conclusion, however, has it the other way around. So this option would weaken the argument because it says that the reasoning is flawed (i.e. backwards).
Dana,

Could you please explain how to arrive at the conclusion "wealth generated by the use of resources can be used for improving the environment."


Thanks in advance.

User avatar
Master | Next Rank: 500 Posts
Posts: 123
Joined: Tue Nov 23, 2010 7:18 pm
Location: Mumbai, India
Thanked: 5 times
Followed by:4 members

by anirudhbhalotia » Mon Dec 06, 2010 1:04 am
A !


In the passage it says -

environmental policies that
excessively restrict the use of natural resources
may diminish the wealth necessary to adopt and
sustain the policies that brought about these
improvements.



The statement in A, clearly suggests and strengthens the same.

(A) Nations sustain their wealth largely through
industrial use of the natural resources found
within their boundaries.

Junior | Next Rank: 30 Posts
Posts: 16
Joined: Tue Nov 30, 2010 2:50 pm

by kt00381n » Fri Dec 10, 2010 6:38 pm
Clear A

Legendary Member
Posts: 2330
Joined: Fri Jan 15, 2010 5:14 am
Thanked: 56 times
Followed by:26 members

by mundasingh123 » Sat Dec 11, 2010 1:19 am
DanaJ wrote:Received a PM.
The reasoning goes as follows:
- there have been notable improvements in the quality of the environment
- this supports the idea that you still need to have some economical activity to support the costs associated with environmental improvements
The conclusion here is that wealth generated by the use of resources can be used for improving the environment.
E - this is actually the "reverse argument" option. This option states that concerns for the environment lead to greater wealth. The conclusion, however, has it the other way around. So this option would weaken the argument because it says that the reasoning is flawed (i.e. backwards).
Hi DanaJ,
Firstly , I think the conclusion is something different .
The columnist concludes that there is credibility in claims of people argue that environmental policies that
excessively restrict the use of natural resources
may diminish the wealth necessary to adopt and
sustain the policies that brought about these
improvements.
Your conclusion " that wealth generated by the use of resources can be used for improving the environment. " may serve as a subconlusion on the basisof which a bigger conclusion may be reached.
Please correct me in case i am wrong.
can you please explain why E is wrong.I didnt get the "reverse argument " funda.

User avatar
Site Admin
Posts: 2567
Joined: Thu Jan 01, 2009 10:05 am
Thanked: 712 times
Followed by:550 members
GMAT Score:770

by DanaJ » Sat Dec 11, 2010 9:49 am
I feel like the two conclusions outlined by you are not sufficiently different to warrant a huge discussion. Both have their merits: yours is more about sticking to the exact wording of the argument (which is very important in CR, for sure!), while mine kind of sees past the mentioning of "people who claim this and that" and refers to the general perception that wealth generated by the use resources helps improve the environment. I feel like they're strikingly similar and picking one or the other does not affect in any way the argument as a whole.

Regarding E: our argument states (either as a conclusion or as a subconclusion, whichever way you feel is best) that:

wealth generated by utilizing resources -------> use it to improve the environment

Option E is the reverse argument in the sense that it goes like so:

concern for improving the environment ------> wealth

As you can see, the two do not match, i.e. E changes the sequence of cause - consequence. In the argument, the wealth is the cause for improvements in environment, while in E the environment is the cause for wealth.

Hope this makes sense!

Legendary Member
Posts: 2330
Joined: Fri Jan 15, 2010 5:14 am
Thanked: 56 times
Followed by:26 members

by mundasingh123 » Sat Dec 11, 2010 10:38 am
DanaJ wrote: Regarding E: our argument states (either as a conclusion or as a subconclusion, whichever way you feel is best) that:

wealth generated by utilizing resources -------> use it to improve the environment

Option E is the reverse argument in the sense that it goes like so:

concern for improving the environment ------> wealth

As you can see, the two do not match, i.e. E changes the sequence of cause - consequence. In the argument, the wealth is the cause for improvements in environment, while in E the environment is the cause for wealth.

Hope this makes sense!
Thanks Dana for explaining this so clearly.

User avatar
Senior | Next Rank: 100 Posts
Posts: 32
Joined: Thu Jan 07, 2010 6:45 am
Thanked: 2 times
GMAT Score:710

by maddy2u » Mon Dec 20, 2010 9:53 am
Answer IMO is A

Newbie | Next Rank: 10 Posts
Posts: 1
Joined: Wed Jan 12, 2011 9:44 pm

by purplespen » Sat Jan 22, 2011 10:52 pm
IMO A

Junior | Next Rank: 30 Posts
Posts: 12
Joined: Thu Mar 17, 2011 8:25 am

by bohemian86 » Tue Apr 26, 2011 9:51 pm

Junior | Next Rank: 30 Posts
Posts: 17
Joined: Wed Jun 15, 2011 11:38 am
Thanked: 1 times

by tailoc » Mon Jul 04, 2011 8:53 am
A is correct because A explain "diminish wealth", causing by environmental policies.
also, C weaken reasoning; B and D,E is true, but isn't mentioned in premises.